文档库 最新最全的文档下载
当前位置:文档库 › 希尔国际商务课后习题参考答案英文版

希尔国际商务课后习题参考答案英文版

希尔国际商务课后习题参考答案英文版
希尔国际商务课后习题参考答案英文版

Global business today

Chapter 1: Globalization

1. Describe the shifts in the world economy over the last 30 years. What are the implications of these shifts for international businesses based in Great Britain? North America? Hong Kong? Answer: The world economy has shifted dramatically over the past 30 years. As late as the 1960s, four stylized facts described the demographics of the global economy. The first was U.S. dominance in the world economy and world trade. The second was U.S. dominance in the world foreign direct investment picture. Related to this, the third fact was the dominance of large, multinational U.S. firms in the international business scene. The fourth was that roughly half of the globe - the centrally planned economies of the Communist world - was off-limits to Western international businesses. All of these demographic facts have changed. Although the U.S. remains the world's dominant economic power, its share of world output and world exports have declined significantly since the 1960s. This trend does not reflect trouble in the U.S. economy, but rather reflects the growing industrialization of developing countries such as China, India, Indonesia, and South Korea. This trend is also reflected in the world foreign direct investment picture. As depicted in Figure 1.2 in the textbook, the share of world output (or the stock of foreign direct investment) generated by developing countries has been on a steady increase since the 1960s, while the share of world output generated by rich industrial countries has been on a steady decline. Shifts in the world economy can also be seen through the shifting power of multinational enterprises. Since the 1960s, there have been two notable trends in the demographics of the multinational enterprise. The first has been the rise of non-U.S. multinationals, particularly Japanese multinationals. The second has been the emergence of a growing number of small and medium-sized multinationals, called mini-multinationals. The fall of Communism in Eastern Europe and the republics of the former Soviet Union have brought about the final shift in the world economy. Many of the former Communist nations of Europe and Asia seem to share a commitment to democratic politics and free market economies. Similar developments are being observed in Latin America. If these trends continue, the opportunities for international business may be enormous. The implications of these shifts are similar for North America and Britain. The United States and Britain once had the luxury of being the dominant players in the world arena, with little substantive competition from the developing nations of the world. That has changed. Today, U.S. and British manufacturers must compete with competitors from across the world to win orders. The changing demographics of the world economy favor a city like Hong Kong. Hong Kong (which is now under Chinese rule) is well located with easy access to markets in Japan, South Korea, Indonesia, and other Asian markets. Hong Kong has a vibrant labor force that can compete on par with the industrialized nations of the world. The decline in the influence of the U.S. and Britain on the global economy provides opportunities for companies in Hong Kong to aggressively pursue export markets.

2. "The study of international business is fine if you are going to work in a large multinational enterprise, but it has no relevance for individuals who are going to work in smaller firms." Evaluate this statement.

Answer: People who believe in this view, and the firms that they work for, may find that they do not achieve their full potential (at best) and may ultimately fail because of their myopia. As barriers to trade decrease and state of the art technological developments take place throughout the world, new opportunities and threats exist on a worldwide basis. The rise of the mini-multinationals suggests there are global opportunities even for small firms. But staying attuned to international markets is not only important from the perspective of seeking profitable opportunities for small firms; it can also be critical for long-term competitive survival. Firms from other countries may be developing products that, if sold internationally, may wipe out small domestic competitors. Scanning international markets for the best suppliers is also important for small firms, for if a domestic competitor is able to tap into a superior supplier from a foreign country, it may be able to seriously erode a small firm's competitive position before the small firm understands the source of its competitor's competitive advantage and can take appropriate counter actions.

3. How have changes in technology contributed to the globalization of markets and of production? Would the globalization of production and markets have been possible without these technological changes?

Answer: Changes in technology have contributed to the globalization of markets and of production in a very substantive manner. For instance, improvements in transportation technology have paved the way for companies like Coca-Cola, Levi Strauss, Sony and McDonalds to make their products available worldwide. Similarly, improvements in communications technology have had a major impact. The ability to negotiate across continents has been facilitated by improved communications technology, and the rapidly decreasing cost of communications has lowered the expense of coordinating and controlling a global corporation. Finally, the impact of information technology has been far reaching. Companies can now gain worldwide

exposure simply by setting up a homepage on the Internet. This technology was not available just a few short years ago. The globalization of production and markets may have been possible without improvements in technology, but the pace of globalization would have been much slower. The falling cost of technology has made it affordable for many developing nations, which has been instrumental in helping these nations improve their share of world output and world exports. The inclusion of these nations, such as China, India, Thailand, and South Korea, has been instrumental in the globalization of markets and production. In addition, improvements in global transportation and communication have made it relatively easy for business executives from different countries to converse with one another. If these forms of technology, including air-travel, fax capability, e-mail, and overnight delivery of packages were not available, it would be much more difficult for businesses to conduct international trade.

4. "Ultimately, the study of international business is no different from the study of domestic business. Thus, there is no point in having a separate course on international business." Evaluate this statement.

Answer: This statement reflects a poor understanding of the unique challenges involved in international business. Managing an international business is different from managing a purely domestic business for at least four reasons. These are: (1) countries are different; (2) the range of problems confronted by a manager in an international business is wider and the problems themselves more complex than those confronted by a manager in a domestic business; (3) an international business must find ways to work within the limits imposed by government intervention in the international trade and investment system; and (4) international transactions involve converting money into different currencies. As a result of these differences, there are ample reasons for studying international business as a specific field of study or discipline.

5. How might the Internet and the associated World Wide Web impact international business activity and the globalization of the world economy?

Answer: According to the text, the Internet and World Wide Web (WWW) promise to develop into the information background of tomorrow's global economy. This improved technology will not only make it easier for individuals and companies in different countries to conduct business with one another, but will also further decrease the cost of communications. These improvements will undoubtedly hasten the already rapid pace of globalization. Another distinct attribute of the Internet and the WWW is that they act as an equalizer between large (resource rich) and small (resource poor) firms. For instance, it does not cost any more for a small software firm to gain visibility via the WWW than it does for a large software company like Microsoft. As a result, the WWW helps small companies reach the size of audience that was previously only within the reach of large, resource rich firms.

6. If current trends continue, China may emerge as the world's largest economy by 2020. Discuss the possible implications of such a development for(1) The world trading system.(2)The world monetary system.(3)The business strategy of today's European and U.S. based global corporations.

(4)Global commodity prices.

Answer: The world trading system would clearly be affected by such a development. Currently China enjoys a somewhat privileged status within the World Trade Organization as a ―developing‖ country. Such a rise to eminence, however, would clearly force it to become a full and equal member, with all the rights and responsibilities. China would also be in a position to actively affect the terms of trade between many countries. On the monetary front, one would expect that China would have to have fully convertible and trading currency, and it could become one of the ―benchmark‖currencies of the world. From the perspective of Western global firms, China would represent both a huge market, and potentially the home base of some very capable competitors. Finally, commodity prices would probably fall.

7. Read the Country Focus in this chapter on the Ecuadorian rose industry, the answer the following questions:

a) How has participation in the international rose trade helped Ecuador’s economy and its people? How has the rise of Ecuador as a center for rose growing benefited consumers in developed nations who purchase the roses? What do the answers to these questions tell you about the benefits of international trade?

b) Why do you think that Ecuador’s rose industry only began to take of 20 years ago? Why do you think it has grown so rapidly?

c) To what extent can the alleged health problems among workers in Ecuador’s rose industry be laid at the feet of consumers in the developed world and their desire for perfect Valentine’s Day roses? d) Do you think governments in the developed world should place trade sanctions on Ecuador roses if reports of health issues among Ecuadorian rose workers are verified? What else might they do to improve the situation in Ecuador?

a) Ecuador is the world’s fourth largest producer of roses. In fact, roses represent the country’s fifth

largest export. The industry is vital to Ecuador’s economy, creating tens of thousands of jobs, jobs that pay significantly above the country’s minimum wage. Taxes and revenues from the rose growers have also helped to pave roads, build schools, and construct sophisticated irrigation systems. For Ecuador’s consumers, the success of the industry means bigger, more vibrant flowers than were previously available. Most students will recognize that the conditions in Ecuador give the country a distinct advantage in rose growing, and that by focusing on the industry, Ecuador has been able to benefit from international trade. Consumers in other countries also benefit from Ecuador’s flower exports with better products.

b) Ecuador’s rose industry began some 20 years ago, and has been expanding rapidly ever since then. Most students will probably focus advances in technology as a key to the industry’s success. Roses are a very fragile, perishable product. Modern technology enables growers to used refrigerated air transport to get the product to markets around the globe. Without that ability, the growers would be limited to the market immediately surrounding the country.

c) Students will probably be divided on this issue with some students arguing that consumers are to blame for the problems, and others placing the blame on the growers. Students taking the first perspective will probably suggest that most consumers purchase their roses with little consideration for how they are grown. Rather, most consumers simply focus on their beauty and price. Students blaming the growers might argue that growers, because they feel the effect of less-than-perfect roses in the form of smaller profits, will be motivated to find ways to produce ever more perfect flowers. Certainly, the use of pesticides and other products can produce a better crop. In the end, there is probably blame on both sides. If more consumers were aware of the health problems resulting from the improper use of pesticides, they would probably demand some changes. Similarly, if pressure were put on the growers to use pesticides safely, health problems could be reduced.

d) Trade sanctions are a tool that is often employed by governments that are making a statement against a specific action or actions. In the case of Ecuador, trade sanctions certainly would be an option, as would publicizing the situation so that more consumers were aware of the conditions. In the end though, it is important to consider the effect of the sanctions or other measures on people like Maria who might lose their livelihood as a result.

Chapter 2: National Differences

1. Free market economies stimulate greater economic growth, whereas state-directed economies stifle growth! Discuss.

Answer: In a market economy, private individuals and corporations are allowed to own property and other assets. This right of ownership provides a powerful incentive for people to work hard, introduce new products, develop better advertising campaigns, invent new products, etc., all in the hopes of accumulating additional personal capital and wealth. In turn, the constant search on the part of individuals and corporation to accumulate wealth enriches the entire economy and creates economic growth. In contrast, in a command economy, private individuals and corporations are not allowed to own substantial quantities of property and other assets. The objective of a command economy is for everyone to work for ―the good of the society.‖Although this sounds like a noble ideal, a system that asks individuals to work for the good of society rather than allowing individuals to build personal wealth does not provide a great incentive for people to invent new products, develop better advertising campaigns, find ways to be more efficient, etc. As a result, command economies typically generate less innovation and are less efficient than market economies.

2. A democratic political system is an essential condition for sustained economic progress. Discuss.

Answer: This question has no clear-cut answer. In the West, we tend to argue that democracy is good for economic progress. This argument is largely predicted upon the idea that innovation is the engine of economic growth, and a democratic political system encourages rather than stifles innovation. However, there are examples of totalitarian regimes that have fostered a market economy and strong property rights protection and experienced rapid economic growth. The examples include four of the fastest growing economies of the past 30 years –South Korea, Taiwan, Singapore, and Hong Kong – all of which have grown faster than Western economies. However, while it is possible to argue that democracy is not a necessary precondition for the establishment of a free market economy, it seems evident that subsequent economic growth leads to establishment of democratic regimes. Several of the fastest-growing Asian economies have recently adopted more democratic governments.

3. What is the relationship between corruption (i.e., bribe taking by government officials) in a country and economic growth? Is corruption always bad?

Answer: Economic evidence suggests that high levels of corruption significantly reduce the economic growth rate in a country. By siphoning off profits, corrupt politicians and bureaucrats reduce the returns to business investment, and hence, reduce the incentive that both domestic and foreign businesses have to invest in that country. The lower level of investment that results has a negative impact on economic growth. However,

while most students will probably agree that corruption is bad, some may point out that the U.S., despite its Foreign Corrupt Practices Act, does allow ―grease payments‖ to expedite or secure the performance of a routine governmental action. According to Congress, ―grease payments‖while technically bribes are not being used to obtain or maintain business, but rather are simply made to facilitate performance of duties that the recipients are already obligated to perform.

4. The Nobel prize-winning economist Amartya Sen argues that the concept of development should be broadened to include more than just economic development. What other factors does Sen think should be included in an assessment of development? How might adoption of Sen’s views influence government policy? Do you think Sen is correct that development is about more than just economic development? Explain.

Answer: Sen has argued that development be assessed less by material output measures such as GNP per capita, and more by the capabilities and opportunities that people enjoy. Sen suggests that development be seen as a process of expanding real freedoms that people experience, and as such, that development requires the removal of major impediments to freedom. Governments influenced by Sen might ensure that basic health care and education programs are available especially for women. Many students will agree with Sen and the notion that development is not just an economic process, but a political one too, and that to succeed citizens must be given a voice in the important decisions made for the country.

5. You are the CEO of a company that has to choose between making a $100 million investment in either Russia or the Czech Republic. Both investments promise the same long-run return, so your choice of which investment to make is driven by considerations of risk. Assess the various risks of doing business in each of these nations. Which investment would you favor and why?

Answer: When assessing the risks of investment, one should consider the political, economic, and legal risks of doing business in either Russia or the Czech Republic. At this time (Fall 2002), the risk in Russia would probably be considered higher than the risk in the Czech Republic. The Czech Republic has just been accepted as a future member of the EU, and as such gains the benefits and stability offered by the EU. Russia, by contrast, is still many years away from even being in a position to be considered by the EU for membership. Depending upon when you are using the book, this situation could be different. (You also may want to substitute other countries into this question depending on current events and the countries with which you feel your students will be most familiar.)

6. Read the Country Focus on India in this chapter and answer the following questions:

a. What kind of economic system did India operate during 1947-1990? What kind of system is it moving towards today? What are the impediments to completing this transformation?

b. How might widespread public ownership of businesses and extensive government regulations have impacted (i) the efficiency of state and private businesses, and (ii) the rate of new business formation in India during the 1947-1990 time frame? How do you think these factors affected the rate of economic growth in India during this time frame?

c. How would privatization, deregulation, and the removal of barriers to foreign direct investment affect the efficiency of business, new business formation, and the rate of economic growth in India during the post-1990 time period?

d. India now has pockets of strengths in key high technology industries such as software and pharmaceuticals. Why do you think India is developing strength in these areas? How might success in these industries help to generate growth in other sectors of the Indian economy?

e. Given what is now occurring in the Indian economy, do you think that the country represents an attractive target for inward investment by foreign multinationals selling consumer products? Why? Answer:

a. The economic system that developed in India after 1947 was a mixed economy characterized by a large number of state-owned enterprises, centralized planning, and subsidies. In 1991, India’s government embarked on an ambitious economic reform program. Much of the industrial licensing system was dismantled, and several areas once closed to the private sector were opened. In addition, investment by foreign companies was welcomed, and plans to start privatizing state-owned businesses were announced. India has posted impressive gains since 1991, however there are still impediments to further transformation. Attempts to reduce import tariffs have been stalled by political opposition from employers, employees, and politicians. Moreover, the privatization program has been slowed thanks to actions taken by the Supreme Court. Finally, extreme poverty continues to plague the country.

b. The mixed economy that developed in India after 1947 was characterized by a large number of state-owned enterprises, centralized planning, and subsidies. This system not only constrained the growth of the private sector, but it also consequently limited the effects of competition that typically promote efficiency and productivity in a free market system. The system even limited the actions of private companies, requiring them to get government approval for routine business activities. Production quotas

and high import tariffs also stunted the development of a healthy private sector, as did restrictive labor laws that made it difficult to fire employees. Foreign exchange restrictions, limitations on foreign investment, controls on land use, and managed prices further exacerbated the situation. It would appear that India’s rate of economic growth was negatively affected during this time frame. By 1994, India’s economy was still smaller than Belgium’s despite having a large population. Both GDP and literacy rates were very low, and some 40 percent of the population lived in poverty.

c. In 1991, India’s government embarked on an ambitious economic reform program. So far, the response to the program has been impressive. The economy expanded at an annual rate of about 6.3 percent from 1994 to 2004. Foreign investment is up from $150 million in 1990 to $6 billion in 2005. Certain sectors of the economy including information technology and pharmaceuticals have done particularly well. Still, problems persist. Actions taken by the government continue to limit efficiency gains for private companies and the country’s high rate of poverty is still a major problem.

d. India’s gains in information technology and pharmaceuticals are impressiv

e. The country has emerged as a vibrant global center for software development, and India’s pharmaceutical companies have taken a strong global position by selling low cost generic versions of drugs that have come of patent in the developed world. As these industries continue to prosper, other sectors of the economy should also see the benefit of spillover effects.

e. Foreign investment is up in India. In fact, foreign investment rose from $150 million in 1990 to $6 billion in 2005. However, whether India is an attractive destination for foreign multinationals selling consumer products remains to be seen. Certainly, the large population will serve to attract some companies, but the fact that some 40 percent of the population is living in abject poverty will scare other companies away. Moreover, it is still not easy to run a company in India thanks to laws limiting everything from who can be fired to who can which products.

Chapter 3: Differences in Culture

1. Outline why the culture of a country influences the costs of doing business in that country. Illustrate your answer with examples.

Answer: Since in a sense the entire chapter is about this question, there can be numerous reasons and examples of how culture influences the costs of doing business. Several are highlighted in the following sentences, but there could be numerous others. When there are simply different norms between how individuals from different countries interact, the costs of doing business rise as people grapple with unfamiliar ways of doing business. For example, while in the US we may get down to business first, and then get to know each other socially later, in many South American countries it is important develop a good social relationship before trying to discuss business issues. Different class structures and social mobility also raise the costs of doing business, for if there are inhibitions against working with people from different classes, then the efficiency with which information can flow may be limited and the cost of running a business increased.

A country's religion can also affect the costs of business, as religious values can affect attitudes towards work, entrepreneurship, honesty, fairness, and social responsibility. In Hindu societies where the pursuit of material well-being can be viewed as making spiritual well being less likely, worker productivity may be lower than in nations with other religious beliefs. Finally, a country's education system can have important implications for the costs of business. In countries where workers receive excellent training and are highly literate, the need for specific worker training programs are decreased and the hiring of additional employees is facilitated.

2. Do you think business practices in an Islamic country are likely to differ from business practices in the United States? If so, how?

Answer: A number of aspects of the cultural differences between an Islamic country and the USA will cause business practices to differ. The role women can take, appropriate etiquette (including simple things like not passing papers with the left hand), holidays, and wining and dining all differ from in the USA. But beyond these, the underlying philosophy and role of business differs from in the USA. Since Muslims are stewards of property for God, rather than owners, they are more likely to use their resources carefully and may be less likely to give up or sell something to a person who may not practice the same stewardship. The importance of fairness to all parties in relations means that over-aggressiveness in self-interest may not be well received, and breaking an agreement, even if technically/legally permissible may be viewed as very inappropriate. Finally, the prohibitions on interest payments in some Islamic countries means that the wording of the terms of an agreement must be done carefully so that "fair profits" are not construed as being "interest payments."

3. What are the implications for international business of differences in the dominant religion of a country?

Answer: Differences in the dominant religion of a country affect relationships, attitudes toward business, and overall economic development. Firstly, differences in religion require inter-cultural sensitivity. This

sensitivity requires things like simply knowing the religious holidays, accepting that some unexpected things may happen "because of Allah's will," or understanding how interpersonal relationships may be different between "believers" and "non-believers." (Hence non-believers may be treated differently.) Secondly, religious beliefs can significantly affect a countries attitude toward business, work, and entrepreneurship. In one country successfully beating a competitor may be considered a great achievement while in another it may be thought of as showing a lack of compassion and disruptive to the society and persons involved, both attitudes that may be derived from underlying religious beliefs. Likewise, hard work may be either rewarded positively or viewed as something of secondary importance to spiritual peace and harmony. Thirdly, different dominant religions may affect the overall competitiveness and potential for economic growth of a nation, and hence attractiveness of a country for international business.

4. Choose two countries that appear to be culturally diverse. Compare the culture of those countries

and then indicate how cultural differences influence

(1)the costs of doing business in each country

(2)the likely future economic development of that country

(3)business practices

(4)business ethics

Answer: Responses to this question will obviously vary based on the countries chosen by the students, and their knowledge of the countries. Hopefully the student can present some information on the dimensions of culture including values, norms, social structure, religion, language, and education of the countries and also describe the key differences and similarities of the countries along these dimensions. Relating the differences between the countries along these dimensions to differences in the costs of doing business, the potential for economic development, and business practices would fully answer the question. (While it may be more difficult for students to come up with really good examples relative to business practices, the costs and prospects for economic development should be quite feasible.)

Chapter 4: Ethics in International Business

1. Review the Management Focus on testing drugs in the developing world and discuss the following questions:

(a) Did Pfizer behave unethically by rushing to take advantage of an epidemic in Nigeria in order to test an experimental drug on sick children? Should the company have proceeded more carefully?

(b) Is it ethical to test an experimental drug on children in emergency settings in the developing world where the overall standard of health care is much lower than in the developed world, and where proper protocols might not be followed?

Answer: Some students might argue that Pfizer was between a rock and a hard place. The company needed to test its drug on children prior to getting FDA approval, yet the company could not find enough sick children to properly test the drug. Most students will probably agree that the company acted irresponsibly with its testing in Nigeria, and that the company should have proceeded more carefully, but some students may also suggest that had the drug proved to be extremely successful in treating bacterial meningitis, Pfizer might have been seen as a hero, despite its questionable ethics.

2. A visiting American executive finds that a foreign subsidiary in a poor nation has hired a 12-year old girl to work on a factory floor, in violation of the company’s prohibition on child labor. He tells the local manager to replace the child and tell her to go back to school. The local manager tells the American executive that the child is an orphan with no other means of support, and she will probably become a street child if she is denied work. What should the American executive do?

Answer: This question, illustrating a potentially very real ethical dilemma facing managers working in foreign subsidiaries, is designed to stimulate class discussion. Students should recognize that neither alternative—violating the company’s position on child labor, nor putting the child out on the streets—seems acceptable. In the end, many students may agree that allowing the child to continue to work in the factory is the lesser of the two evils.

3. Drawing upon John Rawls’ concept, the veil of ignorance, develop an ethical code that will

(a) guide the decisions of a large oil multinational towards environmental protection, and

(b) influence the policies of a clothing company to outsource its manufacturing process?

Answer: According to John Rawls, a decision is just and ethical if people would allow for it when designing a social system under a veil of ignorance. Rawls’ veil of ignorance is a conceptual tool that can contribute towards the moral compass that managers can use to help them navigate through difficult ethical dilemmas. This question can produce some interesting responses particularly in a class with a diverse group of nationalities.

4. Under what conditions is it ethically defensible to outsource production to producers in the

developing world who have much lower labor costs when such actions also involve laying off long term employees in the firm’s home country?

Answer: This question is likely to stimulate some lively discussion, particularly if students have personally felt the impact of this practice. Many American companies are outsourcing not only blue collar work, but white collar positions to the developing world. Students are facing a tenuous job market where positions that they may have sought when they began their college degrees are being ―shipped abroad.‖Some students will argue that companies have to do what is best for all stakeholders, and if that means taking advantage of cheaper labor costs elsewhere, then that is the appropriate strategy. Others however, will probably argue that companies owe a social debt to their home countries, and that loyalty from long term employees should be rewarded.

5. Are facilitating payments ethical?

Answer: Students will probably be divided on this question, and a lively debate should ensue. Certainly, the notion of when in Rome, do as the Romans do. However, those taking this perspective should recognize that it may be difficult to draw the line on exactly what is acceptable under this guise, and when bribery goes too far.

Chapter 5: International Trade Theory

1. “Mercantilism is a bankrupt theory that has no place in the modern world.”Discuss.

Answer: Mercantilism, in its purest sense, is a bankrupt theory that has no place in the modern world. The principle tenant of mercantilism is that a country should maintain a trade surplus, even if that means that imports are limited by government intervention. This policy is bankrupt for at least two reasons. First, it is inconsistent with the general notion of globalization, which is becoming more and more prevalent in the world. A policy of mercantilism will anger potential trade partners because it will exclude their goods from free access to the mercantilist country’s markets. Eventually, a country will find it difficult to export if it imposes oppressive quotas and tariffs on its imports. Second, mercantilism is bankrupt because it hurts the consumers in the mercantilist country. By denying its consumers access to either ―cheaper‖goods from other countries or more ―sophisticated‖ goods from other countries, the mercantilist country’s ordinary consumers suffer.

2. Is free trade fair? Discuss!

Answer: This question is designed to stimulate class discussion. Trade theory tells suggests that specialization and free trade benefits all countries. However, a case can be made in some situations for imposing trade barriers. For example, if a developing country is trying to establish an industry, trade barriers may be needed in the short term until the industry can become competitive. While it could be argued that another country could make the product more efficiently already, is it fair to limit a country’s ability to develop its industrial base?

3. Unions in developed nations often oppose imports from low-wage countries and advocate trade barriers to protect jobs from what they often characterize as “unfair” import competition. Is such competition “unfair?”Do you think that this argument is in the best interests of

(1)the unions

(2)the people they represent

(3)the country as a whole

Answer: The theory of comparative advantage suggests that a country should specialize in producing those goods that it can produce most efficiently, while buying goods that it can produce relatively less efficiently from other countries. Furthermore, the theory suggests that opening a country to free trade stimulates economic growth, which creates dynamic gains from trade. Therefore, it would follow that if low-wage countries can make certain products more efficiently than high wage countries, the low wage countries should produce and export those products. While trade barriers may protect workers and companies, they are a short-term fix at best. Moreover, by protecting industries, the government is not encouraging companies to become more efficient. Instead, they are promoting inefficiency. Consumers lose out because they have higher prices and less choice.

4. What are the potential costs of adopting a free trade regime? Do you think governments should do anything to reduce these costs? What?

Answer: Students will probably be divided on this question, and a lively debate should ensue. For example, certainly, students will probably recognize that by adopting a free trade regime, jobs will be lost in some industries, however they may not agree on exactly what should be done about the jobs losses. Some students might suggest that the government provide retraining programs while others may argue that people lose their jobs everyday and don’t get government assistance to find new ones.

5. Reread the country focus feature on outsourcing service jobs. Is there is a difference between the transference of high paying white collar jobs, such as computer programming and accounting, to developing nations, and low paying blue collar jobs? If so, what is the difference, and should government do anything to stop the flow of white- collar jobs out of the country to countries like India?

Answer: This question is likely to generate a lively debate. Many students will suggest that the outward flow of white-collar jobs is indeed a serious issue, one that should be the focus of government attention. Students taking this perspective are likely to suggest that white-collar jobs are more important to the nation’s future, and that they should remain at home. Other students however, may argue that companies cannot afford to pay the higher wages commanded by white-collar jobs and still remain profitable. Therefore, the argument might be that by taking these jobs outside the country, the company is able to remain viable, and keep other people employed.

6. Drawing upon the new trade theory and Porter’s theory of national competitive advantage, outline the case for government policies that would build national competitive advantage in a particular industry. What kind of policies would you recommend that the government adopt? Are these policies at variance with the basic free trade philosophy?

Answer: Porter’s theory of national competitive advantage argues that four broad attributes of a nation shape the environment in which local firms compete, and that these attributes promote or impede the creation of competitive advantage. These attributes are: factor endowments, demand conditions, related and supporting industries, and firm strategy, structure, and rivalry. Porter goes on to argue that firms are most likely to succeed in industries in which the diamond (which are the four attributes collectively) is favorable. Porter adds two factors to the list of attributes described above: chance and government policy. The New Trade theory addresses a separate issue. This theory argues that due to the presence of substantial scale economies, world demand will support only a few firms in many industries. Underpinning this argument is the notion of first-mover advantages, which are the economic and strategic advantages that accrue to early entrants into an industry. One could argue that when the attributes of a nation are conductive to the production of a product, and when the manufacturers of that product have experienced some ―chance‖events that have provided them first-mover advantages, the governmental policies of that nation should promote the building of national competitive advantage in that particular area. This could be accomplished through government R&D grants, policies that favor the industry in capital markets, policies towards education, the creation of a favorable regulatory atmosphere, tax abatements, and the like. Ask your students whether they think this policy is at variance with the basic free trade philosophy. One could argue that it is, because the government intervention is creating the basis for comparative advantage. Conversely, one could argue that if a country establishes a comparative advantage in a particular area that is based on a unique set of attributes (such as Swiss production of watches), world output will be favorably impacted by letting that country pursue its area of comparative advantage.

7. The world’s poorest countries are at a competitive disadvantage in every sector of their economies. They have little to export. They have no capital; their land is of poor quality; they often have too many people given available work opportunities; and they are poorly educated. Free trade cannot possibly be in the interests of such nations! Discuss.

Answer: This is a difficult question. Certainly, most students will recognize that these countries are in dire straights and need assistance from richer countries. Most students will probably be sympathetic to their cause and suggest various aid programs including education and monetary support to help the countries develop. However, others may be more cautious and promote the notion that assistance would have to come in an organized form with multiple nations working together. The question is an interesting one that should provide students with an eye-opening experience.

Chapter 6: The Political Economy of International Trade

1. Do you think the U.S. government should consider human rights when granting preferential trading rights to countries? What are the arguments for and against taking such a position?

Answer: China is frequently cited as a violator of human rights, and can form the basis for a discussion of this question. While the answer to the first question clearly is a matter of personal opinion, in stating their opinions, students should consider the following points. Trade with the U.S. is very important to China, as China views the U.S. as an important market. The U.S. is also an important source of certain products. Thus, the U.S. has some leverage with trade when trying to influence China’s human rights policies. For this policy to have much effect, however, other nations important to China must adopt similar policies. Otherwise China will simply choose to work with other countries, and U.S. consumers and producers may be more negatively impact than the Chinese. Another concern with tying MFN status to human rights is that denying MFN may make the human rights situation worse rather than better. By engaging in trade, the

income levels in China will increase, and with greater wealth the people will be able to demand and receive better treatment.

2. Whose interests should be the paramount concern of government trade policy - the interests of producers (businesses and their employees) or those of consumers?

Answer: The long run interests of consumers should be the primary concern of governments. Unfortunately consumers, each of whom may be negatively impacted by only a few dollars, are less motivated and effective lobbyists than a few producers that have a great deal at stake. While in some instances it could be argued that domestic consumers will be better off if world-class domestic producers are nurtured and allowed to gain first mover advantages in international markets, it is doubtful that the government will be better than international capital markets at "picking winners", and will more likely pick the firms with the greatest political clout. While employees may well lose jobs if there are more efficient foreign competitors, some would argue that this is just the nature of competition, and that the role of government should be to help these employees get jobs where they can be efficiently employed rather than to protect them from reality in inefficient firms.

3. Given the arguments relating to the new trade theory and strategic trade policy, what kind of trade policy should business be pressuring government to adopt?

Answer: According to the textbook, businesses should urge governments to target technologies that may be important in the future and use subsidies to support development work aimed at commercializing those technologies. Government should provide export subsidies until the domestic firms have established first mover advantages in the world market. Government support may also be justified if it can help domestic firms overcome the first-mover advantages enjoyed by foreign competitors and emerge as viable competitors in the world market. In this case, a combination of home market protection and export-promoting subsidies may be called for.

4. You are an employee of an U.S. firm that produces personal computers in Thailand and then exports them to the U.S. and other countries for sale. The personal computers were originally produced in Thailand to take advantage of relatively low labor costs and a skilled workforce. Other possible locations considered at that time were Malaysia and Hong Kong. The U.S. government decides to impose punitive 100 percent ad valorem tariffs on imports of computers from Thailand to punish the country for administrative trade barriers that restrict U.S. exports to Thailand. How should your firm respond? What does this tell you about the use of targeted trade barriers? Answer: As long as the manufacturing requirements haven't changed significantly, looking at Malaysia or Hong Kong again for production would appear obvious. By the U.S. government introducing a specific ad valorem tariff on Thai computer imports, it would be easy to get around these by looking at other locations. Hence such targeted trade barriers can often be easily circumvented without having to locate production facilities in an expensive country like the U.S.

Chapter 7: Foreign Direct Investment

1. In 2004, inward FDI accounted for some 24 percent of gross capital formation in Ireland, but only 0.6 percent in Japan. What do you think explains this difference in FDI inflows into the two countries?

Answer: Gross capital formation summarizes the total amount of capital invested in factories, stores, office buildings, and so on. When capital investment is high, a country has more favorable growth prospects. The difference between the rates of gross capital formation in Ireland and Japan would indicate that FDI is an important source of investment capital and economic growth in Ireland, but not in Japan. There are several reasons for this. Companies may perceive that Ireland is more attractive as a destination for their investments, or that it is easier to establish operation in Ireland than in Japan. Investors may be cautious about Japan because of its reputation for burdensome regulations.

2. Compare and contrast these explanations of horizontal FDI: the market imperfections approach, Vernon’s product life cycle theory, and Knickerbocker’s theory of FDI. Which theory do you think offers the best explanation of the historical pattern of horizontal FDI? Why?

Answer: Internalization theory seeks to explain why firms often prefer foreign direct investment to licensing as a strategy for entering foreign markets. According to internationalization theory, licensing has three major drawbacks as a strategy for exploiting foreign market opportunities: licensing may result in a firm giving away proprietary technology, licensing does not permit a firm to maintain tight control over its activities, and licensing is not appropriate when a firm’s competitive advantage is based not so much on its products as on the management, marketing, and manufacturing capabilities that produce those products. Vernon’s product life cycle theory argues that firms undertake FDI at particular stages in the life cycle of a

product they have pioneered. They invest in other advanced countries when local demand in those countries grows large enough to support local production. They subsequently shift production to developing countries when product standardization and market saturation give rise to price competition and cost pressures. Investment in developing countries, where labor costs are lower, is seen as the best way to reduce costs.

Finally, Knickerbocker’s theory of FDI suggests that firms follow their domestic competitors overseas. This theory had been developed with regard to oligopolistic industries. Imitative behavior can take many forms in an oligopoly, including FDI.

The second part of this question is designed to stimulate classroom discussion and/or force students to think through these theories and select the one that they feel provides the best explanation for the historic pattern of FDI.

3. Read the opening case on Starbucks, then answer the following questions:

Initially Starbucks expanded internationally by licensing its format to foreign operators. It soon became disenchanted with this strategy. Why?

Why do you think Starbucks has now elected to expand internationally primarily through local joint ventures, to whom it licenses its format, as opposed to a pure licensing strategy?

What are the advantages of a joint venture entry mode for Starbucks over entering through wholly owned subsidiaries? On occasion, Starbucks has chosen a wholly owned subsidiary to control its foreign expansion (e.g. in Britain and Thailand). Why?

Which theory of FDI best explains the international expansion strategy adopted by Starbucks? Answer: Starbucks began its international expansion in Japan where it licensed its formula to a joint venture formed with a local company. Starbucks feared that a pure licensing agreement would not provide it with the control it felt was necessary to successfully replicate the look, feel, and experience of an American Starbucks. To help ensure continuity between its American stores, and its Japanese locations, Starbucks transferred American employees to the Japanese stores to help train workers in the Starbucks way. Using joint ventures has allowed Starbucks to share the cost and risk of developing its foreign markets. While a wholly owned subsidiary would give Starbucks complete control, it also implies that Starbucks would incur all of the cost and risk involved. In Britain, Starbucks did acquire an existing coffee chain that was modeled after Starbucks. Because the chain was already successful, some of the risk that would normally be associated with introducing a new concept to a foreign market was eliminated. Starbucks also shifted to a wholly owned operation in Thailand after its joint venture there experienced difficulty raising capital for further expansion. By acquiring the joint venture, Starbucks was able to gain control over the process. Internalization theory suggests that when licensing is difficult, foreign direct investment is appropriate. Starbucks seems to have followed this philosophy.

4. Compare and contrast these explanations of vertical FDI: the strategic behavior approach and the market imperfections approach. Which theory do you think offers the better explanation of the historical pattern of vertical FDI? Why?

Answer: The strategic behavior approach suggests that FDI flows reflect strategic rivalry between firms in the global marketplace. According to the theory, firms in oligopolistic industries will be motivated to respond to competitor actions with similar actions of their own. Studies show that in the 1950s and 60s, American firms in oligopolistic industries imitated each other’s FDI. Similarly, FDI undertaken by Japanese firms in the 1980s was imitative. The market imperfections theory tries to explain why the first firm in an oligopoly chooses investment over exporting or licensing. Many economists prefer this approach because it addressed the question of whether FDI is more efficient than exporting or licensing for international expansion.

5. You are the international manager of a US business that has just invented a revolutionary new personal computer that can perform the same functions as PCs, but costs only half as much to manufacture. Your CEO has asked you to decide how to expand into the European Union market. Your options are

(1)to export from the United States

(2)to license a European firm to manufacture and market the computer in Europe

(3)to set up a wholly owned subsidiary in Europe

Evaluate the pros and cons of each alternative and suggest a course of action to your CEO. Answer: In considering expansion into the European Union, three options will be considered: FDI, licensing, and export. With export, assuming there are no trade barriers, the key considerations would likely be transport costs and localization. While transport costs may be quite low for a relatively light and high value product like a computer, localization can present some difficulties. Power requirements, keyboards, and preferences in model all vary from country to country. It may

be difficult to fully address these localization issues from the United States, but not entirely infeasible. Since there are many computer manufacturers and distributors in Europe, there are likely to be a number of potential licensees. But by signing up licensees, valuable technological information may have to be disclosed, and the competitive advantage lost if the licensees use or disseminate this information. FDI (setting up a wholly owned subsidiary) is clearly the most costly and time consuming approach, but the one that best guarantees that critical knowledge will not be disseminated and that localization can be done effectively. Given the fast pace of change in the personal computer industry, it is difficult to say how long this revolutionary new computer will retain its competitive advantage. If the firm can protect its advantage for a period of time, FDI may pay off and help assure that no technological know-how is lost. If, however, other firms can copy or develop even superior products relatively easily, than licensing, while speeding up knowledge dissemination, may also allow the firm to get the quickest large scale entry into Europe and make as much as it can before the advantage is lost.

Chapter9 The Foreign Exchange Market

CRITICAL THINKING AND DISCUSSION QUESTIONS

1.The interest rate on South Korean government securities with one-year maturity is 4 percent and

the expected inflation rate for the coming year is 2 percent. The US interest rate on government securities with one-year maturity is 7 percent and the expected rate of inflation is 5 percent. The current spot exchange rate for Korea won is $1 = W1200. Forecast the spot exchange rate one year from today. Explain the logic of your answer.

Answer:The international Fisher effect suggests that the exchange rate will change in an equal amount but in an opposite direction to the difference in nominal interest rates. Hence since the nominal interest rate is 3% higher in the US than in South Korea, the dollar should depreciate by 3% relative to the South Korean Won. Using the formula from the book: (S1 - S2)/S2 x 100 = i$ - iWon and substituting 7 for i$, 4 for iWon, and 1200 for S1, yields a value for S2 of $1=W1165. 2. Two countries, Britain and the US produce just one good: beef. Suppose that the price of beef in the US is $2.80 per pound, and in Britain it is £3.70 per pound.

(a) According to PPP theory, what should the $/£ spot exchange rate be?

(b) Suppose the price of beef is expected to rise to $3.10 in the US, and to £4.65 in Britain. What should be the one year forward $/£ exchange rate?

(c) Given your answers to parts (a) and (b), and given that the current interest rate in the US is 10 percent, what would you expect current interest rate to be in Britain?

?(a) According to PPP, the $/£ rate should be 2.80/3.70, or .76$/£.

?(b) According to PPP, the $/£one year forward exchange rate should be 3.10/4.65, or .67$/£.

?(c) Since the dollar is appreciating relative to the pound, and given the relationship of the international Fisher effect, the British must have higher interest rates than the US. Using the formula (S1 - S2)/S2 x 100 = i£ - i$ we can solve the equation for i£, with S1=.76, S2=.67, i$ = 10, yielding a value of 23.4% for the British interest rates.

?

3. You manufacture wine goblets. In mid June you receive an order for 10,000 goblets from Japan. Payment of ¥400,000 is due in mid December. You expect the yen to rise from its present rate of $1=¥130 to $1=¥100 by December. You can borrow yen at 6% per annum. What should you do?

?(1) The simplest solution would be to just wait until December, take the ¥400,000 and convert it at the spot rate at that time, which you assume will be $1=¥100.

?In this case you would have $4,000 in mid-December.

?(2) Forward Contract

?If the current 180-day forward rate is lower than 100¥/$, then a forward contract might be preferable since it both locks in the rate at a better level and reduces risk.

?If the rate is above ¥100/$, then whether you choose to lock in the forward rate or wait and see what the spot does will depend upon your risk aversion.

?(3) Borrow Money

?There is a third possibility also. You could borrow money from a bank that you will pay back with the ¥400,000 you will receive (400,000/1.03 = ¥388,350 borrowed),

convert this today to US$ (388,350/130 = $2,987), and then invest these dollars in a

US account. For this to be preferable to the simplest solution, you would have to be

able to make a lot of interest (4,000 - 2,987 = $1,013), which would turn out to be an

annual rate of 51% ((1,013/4000) * 2). If, however, you could lock in these interest

rates, then this method would also reduce any exchange rate risk. What you should

do depends upon the interest rates available, the forward rates available, how large a

risk you are willing to take, and how certain you feel that the spot rate in December

will be ¥100 = $1.

4. You are CFO of a U.S. firm whose wholly owned subsidiary in Mexico manufactures component parts for your U.S. assembly operations. The subsidiary has been financed by bank borrowings in the United States. One of your analysts told you that the Mexican peso is expected to depreciate by 30 percent against the dollar on the foreign exchange markets over the next year. What actions, if any, should you take?

?Your financing and operating capital are in dollars, yet many of your costs (labor) must be in peso. Your hard assets are all in peso, and their value will decline. On the other hand, if the peso depreciates, then your dollars will go further. So perhaps doing nothing is the best approach. If you are pretty sure that the peso will depreciate, then you may want to avoid any major peso-denominated costs that you can until after devaluation. That may mean holding back on shipments if possible, and you may want any dollar-denominated purchases made before the devaluation. You may want to move any peso-denominated major accounts into dollars before the devaluation.

Chapter 11: The Strategy of International Business

1. In a world of zero transportation costs, no trade barriers, and non-trivial differences between nations with regard to factor endowments, firms must expand internationally if they are to survive. Discuss.

Answer: Given differences in countries with respect to factor endowments, the theory of comparative advantage suggests that different activities should take place in the countries that can perform them most efficiently. If there are also no barriers or costs to trade, then it is likely that a lot of industries will be based out of the countries that provide the best set of factor endowments. For a firm that is located in a sub-optimal location, it will either have to expand internationally or switch to a different industry where the factor endowments are in its favor. For firms already located in the countries with the most favorable factor endowments for their industry, however, there may not be a need to expand internationally. Firstly, the firm may be content to simply focus on the domestic market. But if the firm does want to expand internationally, it may be able to do so via licensing or exporting, and need not necessarily undertake FDI. Thus, not only in theory, but also in practice many firms are able to survive quite well without having to expand internationally.

2. Plot the position of the following firms on Figure 11.8 - Procter & Gamble, IBM, Nokia, Coca-Cola, Dow Chemical, US Steel, and McDonald's. In each case justify your answer.

Answer: Procter & Gamble would be located in the middle right-hand portion of the graph. This is a position of high pressures for local responsiveness and moderate pressures for cost reductions. P&G sells personal and home care products, which do face pressures for local responsiveness. Although these products are not commodities, there are many competitors in P&G industries, which implies a moderate degree of cost pressures. IBM would be in the upper middle portion of the graph. This is a position of moderate pressure for local responsiveness and high pressure for cost reductions. There is a moderate amount of pressure for local responsiveness for IBM products, due to language differences and differing voltage requirements for

electronic products across countries. IBM is in a very competitive industry, and cost pressures are high. Nokia manufactures wireless handsets and infrastructures such as switches. Nokia, because it must customize its product offering according to the technical standards prevailing in a given country would be in the lower right hand side of the graph. Coca-Cola is a commodity type product, and it would be located in the upper left-hand portion of the graph. This is a position of low pressures for local responsiveness and high pressures for cost reductions. Dow Chemical and U.S. Steel would both be located in the upper left-hand portion of the graph. Both Dow and U.S. Steel sell products that are commodity-like by nature. As a result, cost pressures would be high and local responsiveness pressures would be low for these products. Finally, McDonalds would be located in the middle left-hand portion of the graph. Pressures for local responsiveness would be low, and cost reduction pressures would be moderate. McDonalds sells a semi commodity-like product, but not to the same degree as Dow Chemical of U.S. Steel.

3. Re-Read the Management Focus box on Procter & Gamble and then answer the following questions:

a) What strategy was Procter & Gamble pursuing when it first entered foreign markets in the period up until the 1980s?

b) Why do you think this strategy became less viable in the 1990s?

c) What strategy does Procter & Gamble appear to be moving toward? What are the benefits of this strategy? What are the potential risks associated with it?

Answer: Procter & Gamble initially expanded internationally when it entered Canada in 1915. However, even after expanding into Western Europe and Asia in the 1960s and 1970s, the company sill maintained all product development at its headquarters location in Cincinnati, Ohio. Subsidiary units were responsible for manufacturing, marketing, and distributing the products in their local markets. However, by the 1990s several factors caused Procter & Gamble to reconsider its international strategy. Barriers to low-cost trade were falling rapidly worldwide, and fragmented national markets were merging into larger regional or global markets. In addition, the retailers through which the company distributed its products were growing larger and more global, and were demanding price discounts from Procter & Gamble. The company now appears to be moving towards a transnational strategy in which there are seven self-contained business units, each responsible for the complete generation of profits from its products, and for manufacturing, marketing, and development. A transnational strategy is complex, and the company will have to balance the demands of responding to local market needs for its consumer products, while at the same time reaching its cost savings goals.

4. What do you see as the main organizational problems that are likely to be associated with the implementation of a transnational strategy?

Answer: Simultaneously trying to achieve cost efficiencies, global learning, and local responsiveness places difficult and contradictory demands on an organization. Managing these conflicting demands requires the setting of control and motivational policies for people and organizations that force balancing of these demands at multiple levels within firms. The organizational challenges involve managing these inherent conflicts to resolutions that serve the best interests of the firm overall.

5. Reread the Management Focus box on the alliance between Cisco and Fujitsu. What are the benefits to Cisco and Fujitsu respectively of the alliance? What are the risks to Cisco? How can Cisco mitigate those risks?

Answer: The Cisco-Fujitsu venture was important to both companies as they develop the next generation of routers. The firms will be able to pool their R&D efforts, share complementary technology, and get products to market more quickly. The two companies also believe that the combination of Cisco’s technology together with Fujitsu’s production expertise will enable them to produce more reliable products. In addition, the alliance provides Cisco with access to the Japanese market, a market that it believes will be important in the future, and a more complete product line for Fujitsu. Cisco will have to ensure that it puts proper safeguards in place as it shares its technology with Fujitsu, or risk creating a competitor.

Chapter 12: Entering Foreign Markets

1. Review the Management Focus on ING. ING chose to enter the U.S. financial services market via acquisitions rather than greenfield ventures. What do you think are the advantages to ING of doing this? What might the drawbacks be? Does this strategy make sense? Why?

Answer: Most students will probably agree that ING’s strategy of acquiring firms with a strong local presence makes sense. The company maintains the local management team and products, yet sells its own ING products as well. This strategy allows the company to act locally, while building a global name.

2. Licensing propriety technology to foreign competitors is the best way to give up a firm's

competitive advantage. Discuss.

The statement is basically correct - licensing proprietary technology to foreign competitors does significantly increase the risk of losing the technology. Therefore licensing should generally be avoided in these situations. Yet licensing still may be a good choice in some instances. When a licensing arrangement can be structured in such a way as to reduce the risks of a firm's technological know-how being expropriated by licensees, then licensing may be appropriate. A further example is when a firm perceives its technological advantage as being only transitory, and it considers rapid imitation of its core technology by competitors to be likely. In such a case, the firm might want to license its technology as rapidly as possible to foreign firms in order to gain global acceptance for its technology before imitation occurs. Such a strategy has some advantages. By licensing its technology to competitors, the firm may deter them from developing their own, possibly superior, technology. And by licensing its technology the firm may be able to establish its technology as the dominant design in the industry. In turn, this may ensure a steady stream of royalty payments. Such situations apart, however, the attractions of licensing are probably outweighed by the risks of losing control over technology, and licensing should be avoided

3. Discuss how the need for control over foreign operations varies with firms’strategies and core competencies. What are the implications for the choice of entry mode?

If a firm’s competitive advantage (its core competence) is based on control over proprietary technological know-how, licensing and joint venture arrangements should be avoided if possible so that the risk of losing control over that technology is minimized. For firms with a competitive advantage based on management know-how, the risk of losing control over the management skills to franchisees or joint venture partners is not that great. Consequently, many service firms favor a combination of franchising and subsidiaries to control the franchises within particular countries or regions. The subsidiaries may be wholly owned or joint ventures, but most service firms have found that joint ventures with local partners work best for controlling subsidiaries.

4. A small Canadian firm that has developed some valuable new medical products using its unique biotechnology know-how is trying to decide how best to serve the European Community market. Its choices are given below. The cost of investment in manufacturing facilities will be a major one for the Canadian firm, but it is not outside its reach. If these are the firm’s only options, which one would you advise it to choose? Why?

(1)Manufacture the product at home and let foreign sales agents handle marketing.

(2)Manufacture the products at home but set up a wholly owned subsidiary in Europe to handle marketing.

(3)Enter into a strategic alliance with a large European pharmaceutical firm. The product would be manufactured in Europe by a 50/50 joint venture, and marketed by the European firm.

Answer: If there were no significant barriers to exporting, then option (iii) would seem unnecessarily risky and expensive. After all, the transportation costs required to ship drugs are small relative to the value of the product. Both options (i) and (ii) would expose the firm to less risk of technological loss, and would allow the firm to maintain much tighter control over the quality and costs of the drug. The only other reason to consider option (iii) would be if an existing pharmaceutical firm could also give it much better access to the market and potentially access to its products and technology, and that this same firm would insist on the 50/50 manufacturing joint venture rather than agreeing to be a foreign sales agent. The choice between (i) and (ii) boils down to a question of which way will be the most effective in attacking the market. If a foreign sales agent can be found that is already quite familiar with the market and who will agree to aggressively market the product, the agent may be able to increase market share more quickly than a wholly owned marketing subsidiary that will take some time to get going. On the other hand, in the long run the firm will learn a great deal more about the market and will likely earn greater profits if sets up its own sales force. Chapter 13: Exporting, Importing, and Countertrade

1. A firm based in Washington State wants to export a shipload of finished lumber to the Philippines. The would-be importer cannot get sufficient credit from domestic sources to pay for the shipment but insists that the finished lumber can be quickly resold in the Philippines for a profit. Outline the steps the exporter should take to effect this export to the Philippines.

Answer: The exporter should recommend to the importer that the importer apply to Eximbank for a loan. Eximbank has a direct lending operation under which it lends dollars to foreign borrowers for use in purchasing U.S. exports. The foreign borrowers use the loans to pay U.S. suppliers and repay the loan to Eximbank with interest.

2. You are the assistant to the CEO of a small textile firm that manufactures high-quality, premium-priced, stylish clothing. The CEO has decided to see what the opportunities are for exporting and has asked you for advice as to the steps the company should take. What advice

would you give the CEO?

Answer: This question is designed to stimulate classroom discussion and/or to encourage your students to ―think‖about the export process in completing a written answer for this question. There are a number of approaches that can be pursued in answering this question. The first step might be to tap into some of the government information sources that are available, free of charge, to see if international markets are available for the company’s product. There are also a number of resources on the Internet, mentioned throughout the text that can assist companies in learning about the foreign market potential of their products. Another approach would be to contact an export management company for assistance. While this approach may involve some cost, it may be the fastest way to get ―up and running‖ in regard to initiating an export program.

3. An alternative to using a letter of credit is export credit insurance. What are the advantages and disadvantages of using export credit insurance rather than a letter of credit for exporting (a) a luxury yacht from California to Canada, and (b) machine tools from New York to the Ukraine?

Answer: Exporters prefer to get letters of credit from importers. However, when the importer is in a strong bargaining position and able to play competing suppliers off against each other, an exporter may have to forgo a letter of credit. The lack of a letter of credit exposes the exporter to the risk that the foreign importer will default on payment. The exporter can insure against this possibility by buying export credit insurance. Students may suggest that in the case of the luxury yacht, should the importer fail to make payment, the clearly defined laws of Canada would make it easier to go after the importer than would be the case with the machine tools in the Ukraine, and that therefore a letter of credit is less important for the yacht exporter. On the other hand, students may note that there is probably more competition in machine tools as compared to luxury yachts and that the exporter of machine tools may lose the sale if the exporter insists on a letter of credit.

4. How do you explain the popularity of countertrade? Under what scenarios might its popularity increase still further by the year 2010? Under what scenarios might its popularity decline? Answer: This question requires students to speculate on the future state of global trade. As trade between developing and developed countries, and trade among developing countries continues to grow, many students will predict that the popularity of countertrade will increase by the year 2010. Some students may predict a decline in the popularity of countertrade by 2010 as countries from the former Soviet Union and Eastern European Communist bloc either become members of the EU an adopt the fully convertible euro as their currency, or develop their own fully convertible currency.

5. How might a company make strategic use of countertrade schemes as a marketing weapon to generate export sales revenues? What are the risks associated with pursuing such a strategy? Answer: Counterrtrade is an alternative means of structuring an international sale when conventional means of payment are difficult, costly, or nonexistent. The governments of developing countries sometimes insist on a certain amount of countertrade. Thus, if a firm is unwilling to enter a countertrade agreement, it may lose an export opportunity to a competitor that is willing to make a countertrade agreement. Companies that are willing to entertain countertrade as a means of financing, will have an advantage over those firms that prefer traditional forms of financing. Firms engaging in countertrade must be willing to invest in an in-house trading department dedicated to arranging and managing countertrade deals, and must be aware of the quality of the products received in countertrade deals.

Chapter 14: Global Production, Outsourcing, and Logistics

1. An electronics firm is considering how best to supply the world market for microprocessors used in consumer and industrial electronic products. A manufacturing plant cost approximately $500 million to construct and requires a highly skilled work force. The total value of the world market for this product over the next 10 years is estimated to be between $10 and $15 billion. The tariffs prevailing in this industry are currently low. What kind of manufacturing strategy do you think the firm should adopt - concentrated or decentralized? What kind of location(s) should the firm favor for its plant(s)?

Answer: The firm should pursue a concentrated manufacturing because (1) the tariffs prevailing in the industry are low, (2) the cost of building a plant to produce the microprocessors is high, and (3) the product's value-to-weight ratio is high. All of these factors favor a concentrated vs. a decentralized manufacturing strategy. In terms of location, the company should consider three factors: country factors, technology factors, and product factors. First, in terms of country factors, the firm should locate its plant in a country that has a highly skilled pool of workers available. That criterion probably limits the firm to developed nations. Second, in terms of technology factors, the firm is compelled to limit the number of its manufacturing facilities because of the high cost of constructing a plant. Third, in terms of product factors, the firm can manufacturer its product in a central location due to the relatively high value-weight ratio and the

universal appeal of the product.

2. A chemical firm is considering how best to supply the world market for sulfuric acid. A manufacturing plant costs approximately $20 million to construct and requires a moderately skilled work force. The total value of the world market for this product over the new 10 years is estimated to be between $20 and $30 billion. The tariffs prevailing in this industry are moderate. Should the firm favor concentrated manufacturing or decentralized manufacturing? What kind of location(s) should the firm seek for its plant(s)?

Answer: This question is a tougher call than the scenario depicted in Question #1. The firm should probably pursue a limited decentralized manufacturing strategy (meaning that the firm should not set up a plant in every country that it sells to, but should set up plants in several "regions" of the world). This strategy makes sense because (1) The tariffs prevailing in the industry are moderate (rather than low), (2) the cost of constructing a facility is relatively modest ($20 million), and (3) only a moderately skilled work force is needed (which is probably available in many low-cost regions of the world). The firm should select its location based on country factors, technology factors and product factors. In terms of country factors, the firm should find locations where semi-skilled labor is inexpensive. In terms of technology factors, the firm is not constrained by a high fixed costs associated with its product, so technology is not a pervasive issue. Finally, product factors favor the firm locating in several locations throughout the world. The company's product has a low value-weight ratio, making it unattractive to produce the product in a central location and export it across the world.

3. A firm must decide whether to make a component part in-house or to contract it out to an independent supplier. Manufacturing the part requires a nonrecoverable investment in specialized assets. The most efficient suppliers are located in countries with currencies that many foreign exchange analysts expect to appreciate substantially over the next decade. What are the pros and cons of (a) manufacturing the component in-house and (b) outsourcing manufacture to an independent supplier? Which option would you recommend? Why?

Answer: Manufacturing in-house would reduce the risk of currency appreciation and rising costs from independent suppliers. Specialized asset investment would make firm dependent on specific suppliers, however, technological know-how would be protected, and improved scheduling would be available. Out-sourcing would be beneficial if the product using the component fails in the market because the supplier will bear the cost of the non-recoverable investment, and flexibility in case a better component can be designed or bought would be preserved. Outsourcing would also lower organizational and coordination costs. Based on what we know, manufacturing in house may be slightly preferred, but other information could tip the decision the other way.

4. Explain how an efficient materials management function can help an international business compete more effectively in the global marketplace.

Answer: Given the complexity involved in coordination of material and product flows in a multinational enterprise (purchases, currency exchange, inbound and outbound transportation, production, inventory, communication, expediting, tariffs and duties), a materials management function can help to assure that these flows take place in the most efficient manner possible. A related advantage is that by having a materials management function, a firm may obtain improved information about the costs of different transport alternatives, and choose to reconfigure some of its flows to better take advantage of these costs. By being better able to utilize just in time techniques, the cost of production can be lowered while the quality is increased. The materials management function can also help an international business to develop information technology systems that allow it to better track the flow of goods throughout the firm.

Chapter 15: Global Marketing and R&D

1. Imagine you are the marketing manager for a US manufacturer of disposable diapers. Your firm is considering entering the Brazilian market. Your CEO believes the advertising message that has been effective in the United States will suffice in Brazil. Outline some possible objections to this. Your CEO also believes that the pricing decisions in Brazil can be delegated to local managers. Why might she be wrong?

Answer: While babies’ behinds serve the same function in all cultures, and the product's technical standards may be similar, sensitivity to bodily functions does vary across cultures. Thus the advertising message may need to be changed for different attitudes towards what is appropriate advertising. Likewise, where it might be progressive to show an ad with a male changing a diaper in some countries, in other countries this message could be lost or misinterpreted. Another consideration would be the noise level created by the advertising message of competitor's products, which may well be different in Brazil. While local demand and price elasticity decisions should play an important role in Brazil, pricing should not be left solely to the discretion of the local managers. Since this is a global business, your firm will likely be competing in Brazil with some of the same competitors as elsewhere. Thus pricing decisions in one country can have an impact

on pricing and competition in other markets. Similarly, your firm may want to position and price the brand similarly across different South American countries.

2. Within 20 years we will have seen the emergence of enormous global markets for standardized consumer products. Do you agree with this statement? Justify your answer.

Answer: One could either choose to agree or disagree, while the best answer would likely hedge it somewhere in the middle. There clearly already are enormous global markets already for products like Coke and Levis, while it is questionable whether there will ever be a global consumer market for Norwegian lutefisk. More global consumer markets will likely emerge, but there will continue to be national distinctions for many products.

3. You are the marketing manager of a food products company that is considering entering the Indian market. The retail system in India tends to be very fragmented. Also, retailers and wholesalers tend to have long-term ties with Indian food companies, which makes access to distribution channels difficult. What distribution strategy would you advise the company to pursue? Why?

Answer: The firm should sell to either wholesalers or import agents. Because the retail system in India is very fragmented, it would be very expensive for the firm to make contact with each individual retailer. As a result, it would be more economical for the firm to sell to wholesalers or import agents. Import agents may have long-term relationships with wholesalers, retailers, and/or other import agents. Similarly, wholesalers may have long-standing relationships with retailers and, therefore, be better able to persuade them to carry the firm’s product than the firm itself would.

4. Price discrimination in indistinguishable from dumping. Discuss the accuracy of this statement? Answer: In some specific instances this statement is correct, but as a general rule it is not. When a firm is pricing lower in a foreign country than it is in its domestic market, it can be difficult to distinguish dumping from price discrimination unless it is clear that the firm is selling at below cost in the foreign market. Yet when costs are reasonably well known and all prices are above these, or if the firm is pricing lower in its domestic market than in foreign markets, it can reasonably concluded that price discrimination rather than dumping is occurring.

5. You work for a company that designs and manufactures personal computers. Your company’s R&D center is in North Dakota. The computers are manufactured under contract in Taiwan. Marketing strategy is delegated to the heads of three regional groups: a North American group (based in Chicago), a European group (based in Paris), and an Asian group (based in Singapore). Each regional group develops the marketing approach within its region. In order of importance, the largest markets for your products are North America, Germany, Britain, China, and Australia. Your company is experiencing problems in its product development and commercialization process. Products are late to market, the manufacturing quality is poor, and costs are higher than projected, and market acceptance of new products is less than hoped for. What might be the source of these problems? How would you fix them?

Answer: The dispersion of activities makes sense - products are produced in the lowest cost location and marketed by people familiar with local conditions. (The R&D in North Dakota must be a historical fluke.) Yet this makes the coordination task extremely complex, and information required for successful commercialization is likely not being effectively communicated among all the appropriate people. Greater cross-functional integration in the new product development process should help to improve product development and commercialization.

Chapter 16: Global Human Resource Management

1. What are the main advantages and disadvantages of the ethnocentric, polycentric, and geocentric approaches to staffing policy? When is each approach appropriate?

Answer: The answer to this question is contained in Table 16.1 in the text. An ethnocentric staffing policy is one in which key management positions are filled by parent country nationals. The advantages of the ethnocentric approach are: (1) Overcomes lack of qualified managers in host country, (2) Unified culture, and (3) Helps transfer core competencies. The disadvantages of the ethnocentric approach are: (1) Produces resentment in host country, and (2) Can lead to cultural myopia. An ethnocentric approach is typically appropriate for firms utilizing an international strategy. A polycentric staffing policy requires host country nationals to be recruited to manage subsidiaries, while parent country nations occupy key positions at corporate headquarters. The advantages of the polycentric approach are: (1) Alleviates cultural myopia, and (2) It is inexpensive to implement. The disadvantages of the polycentric approach are: (1) Limits career mobility, and (2) Isolates headquarters from foreign subsidiaries. A polycentric approach is typically appropriate for firms utilizing a localization strategy. A geocentric staffing policy seeks the best people for key jobs throughout the organization, regardless of nationality. The advantages of a geocentric approach

are: (1) Uses human resources efficiently, (2) Helps build strong culture and informal management network. The disadvantages of the geocentric staffing policy are: (1) National immigration policies may limit implementation, and (3) It is expensive to implement. A geocentric approach is typically appropriate for firms unitizing a global or transnational strategy.

2. Research suggests that many expatriate employees encounter problems that limit both their effectiveness in a foreign posting and their contribution to the company when they return home. What are the main causes and consequences of these problems, and how might a firm reduce the occurrence of such problems?

Answer: The primary causes of expatriate problems are the inability of the spouse to adjust, inability of the employee to adjust, other family problems, personal/emotional maturity, and an inability to cope with the larger overseas responsibilities. The consequences of such problems are that an employee can be ineffective or detrimental overseas, and/or may return prematurely before the assigned job tasks are completed. A firm can reduce the occurrence of expatiate problems by developing an effective selection process, training, and repatriation program. The most successful expatriates seem to be those who have high self-esteem and self-confidence, get along well with others, are willing to attempt to communicate in a foreign language, and can empathize with people of other cultures. An expatriate training program should include cultural, language, and practical training. Cultural training seeks to foster an appreciation of the host country's culture so that the expatriate behaves accordingly. Language training involves training in local language both from a business and personal perspective. Practical training is aimed at assisting the expatriate manager and her family to ease themselves into day-to-day life in the host country. The sooner a day-to-day routine is established, the better the prospects are that the expatriate and family will adapt successfully. Before leaving, however, specific plans and procedures should be in place for the repatriation of the employee.

3. What is the link between an international business's strategy and its human resource management policies, particularly with regard to the use of expatriate employees and their pay scale?

Answer: In firms pursuing a localization strategy, a polycentric staffing approach is most common and there are relatively few expatriates or the associated pay issues. Expatriates are more common in firms with international strategies, and an ethnocentric staffing approach is utilized. In this situation the pay is often based on home country levels, with adjustments as required for differing living costs and taxes as outlined by the balance sheet approach. Firms pursuing global standardization or transnational strategies most often use a geocentric approach to staffing, where the best individuals (regardless of nationality) are chosen fill positions in any country. Here the pay issues for expatiates can become particularly complex, as allowance must be made for home country norms, host country costs and expectations, and global norms across the company.

4. In what ways can organized labor constrain the strategic choices of an international business? How can an international business limit these constraints?

Answer: Organized labor can significantly constrain the choices firms make with respect to location. International firms (or domestic ones for that matter) often choose to locate new facilities in places where there is relative labor peace and harmonious working relations. Labor can also raise objections and threaten disruptive behavior if a firm decides to move some activities to other locations - which in some cases only reinforces the need for relocating the activities. Organized labor has also attempted to (i) set-up their own international organizations, (ii) lobby for national legislation to restrict multinationals, and (iii) achieve regulation of multinationals through international organization such as the United Nations. However, none of these broader efforts have been that successful. International businesses have the advantage of being able to provide or take away jobs, and in today's labor market that gives them considerable power. As a condition of opening or expanding a facility, firms can negotiate favorable conditions with local unions and force unions to compete against each other for the gains in membership.

国际商务英语 初级 试卷及参考答案

★绝密·考试前严禁任何人翻阅 全国外经贸从业人员考试 国际商务英语等级考试(初级) 试题册 考试时间:2014年5月31日 09:30-11:30 ※※※※※※※※※※※※※※※※※※※※※※※※※※※※※※※※※※※※※※※※※※※※※※ 考生姓名: 准考证号: 考点(考区): 注意事项 一、考生将姓名、身份证号、考试项目、考点(考区)、准考证号填涂在试题册及答题卡(答 题纸)相应位置。 二、考生在规定考试时间内做完试题册上的试题,并将答案填涂在答题卡(答题纸)相应位 置,写在试题册上的答案一律无效。全部答题时间为120分钟。 三、考生在答题卡上作答时,切忌超出答题区域。如因超出区域作答导致答题卡失效,由考 生个人负责。客观题按题号顺序进行填涂,主观题在每题左上角写清题号按顺序作答。 凡因题号不清导致考试成绩有误,由考生个人负责。 四、考生不得将试题册、答题卡(答题纸)带出考场。考试结束,监考员收卷后考生方可离 开考场。 五、考生注意对自己的答案保密。若被抄袭,一经发现,后果自负。

★请将答案写在答题卡上,答案写在试卷上无效。 一、客观题(本大题有60小题,每小题1分,共60分) [1-20] Listening: 20%(听力,20分) Section A Directions: Listen to the following dialogue and choose the correct word or words to fill each gap from A,B,C or D. This section accounts for 5 points. Each question will be read twice. Mike: It would be nice to see you again, Mary. Are you free on Sunday evening? Mary: I’ m sorry, I’m not. I’ve got to visit my aunt 1 . Mike: Oh, that’s a pity. Monday’s difficult for me. What about Tuesday evening? Mart: Tuesday’s bad for me as well. I’ve got to go to a meeting. Mike: Wednesday then? Mary: No, Wednesday’s out for me, I’m afraid. I’ve got to 2 and do some work. I really must. Mike: Oh, that’s a shame. Well, I can’t 3 Thursday. What about Friday? Mary: I’m 4 sorry. I’ve got to go out for dinner on Friday. Mike: Have you got to? Can’t you get out of it? Mary: I’m afraid not. I’ve simply got to go. Mike: Well, it looks as if we’ll have to wait till next week then. Mary: Yes, I’m sorry, Mike. Look, I must go now. I have to meet Lisa in ten minutes. Ring me next week 5 . Mike: Fine. Try and keep an evening free for me. 1.A. in the hospital B. in hospital C. in hotel D. in hostel 2.A. stay in B. staying C. stay D. sleep 3.A. make that B. make this C. make D. make it 4.A. outright B. awfully C. really D. totally 5.A. some time B. sometimes C. sometime D. some times Section B Directions: Listen to the following passage and choose the correct word or words to fill each gap from A,B,C or D. This section accounts for 5 points. Each question will be read twice. For international investors, the decision to invest in a foreign country, whether to establish or 6 a plant or a sales network, or to purchase stocks or bonds, is a difficult one. Before they make the decision to invest abroad, international investors will have to make sure that the

国际商务函电术语翻译!

2014函电考试题型: I. Translate the following terms into English or Chinese. (15%, 0.5’×30=15) II. Translate the following sentences into Chinese. (15%, 3’×5=15) III. Translate the following sentences into English (25%, 5’×5=25) IV. Fill in the Contract form in English with the particulars given below: (10%) V.Check the following letter of credit with the given contract terms given below and write a letter asking for amendments to the letter of credit. (20%) VI. Write a letter an English letter in a proper form based on the following information.(15%) TERMS 1.Assorted designs 花色图案混合搭配 2.for your information // for your inspection供参考 3.Invitation card请柬 4.in the market for 正在寻购 5.established brand 名牌 6. a well-established exporter信誉良好的出口商 7.rock bottom price 最低价格 8.Principal 委托人本人 9.from ready stock供现货 10.bulk shipment散装运输 11.Promissory note本票 12.商务参赞处Commerce Counsellor’s Office 13.另函by separate post 14.建立贸易关系establish business relations 15.商会Chamber of Commerce 16.在平等互利、互通有无的基础上adhere to the principle of equality and mutual benefit 17.工艺品handicraft 18.进出口import and export 19.以我方为受益人的in our favour 20.试订购trial order 21.推销新产品promote new products 22.精湛的工艺delicate workmanship 23.售后服务after service 24.分三个月每月平均装运in three equal monthly shipment 25.由买方选择at buyer’s option 26.一系列的商务洽谈a sequence of business negotiation 27.以我方最后确认为准subject to our final confirmation 28.销售季节sales season 29.CFR Cost and Freight 30.进口许可证certificate of import license 31.虚盘Non-firm enquiry 32.实盘Firm enquiry 33.商标trademark 34.急需货物imperative articles 35.电汇T/T telegraphic transfer 36.议付negotiation 37.运费freight 38.空运提单air bill of lading 39.租船契约charter party 40.延期装运delay shipment 41.装箱单packing list 42.中性包运neutral packing 43.分批装运partial shipment 44.出票人drawer 45.履行合同fulfill a contract 46.价格差距price spread 47.一式两份in duplicate 48.代表我方索款通知debit note on our behalf (not sure) 49.偷窃提货不着险TPND Theft, Pilferage and Non-Delivery Risks 50.水渍险WPA With Particular Average 51.议付行negotiating bank

国际商务英语综合教程辅导用书

国际商务英语综合教程辅导用书 基本信息 出版社对外经济贸易大学出版社 出版时间20xx/10/01 ISBN9787566304759 亚马逊销量排名第88位 - 图书英语与其他外语大学英语课文辅导 字数10万字 页数139页 简介 《新世界商务英语系列教材:(第2版)》分13章,从国际商务简介、商务组织、经营方式、经济学入门、市场经济、国际贸易实务简介、国际贸易术语及定价原则、营销、营销组合与产品促销、国际支付与结算、外汇汇率到国际商务谈判策略,每章都分门别类介绍该章的专业知识。 目录 第一章国际商务简介(A BriefIntroduction to International Business) 1.什么是商务(What Is。Business) 2.什么是国际商务(What Is International Business)

3.国际商务活动范围(The Scope ofInternationaI:BusinessActivities) 4.国际风险(International Risk) 5.国际商法(International Business Law) 6.商业信用(mercial Credit) 7.国际商务管理(Management of International Business) 8.世界贸易组织简介(A BriefIntroduction to the WTO) 第二章商务组织(Business Organizations) 1.概述(Introduction) 2.独资企业(Sole Proprietorship) 3.一般合伙企业(General Partnership) 4.有限合伙企业(Limited Partnership) 5.合股公司(Joint Stock pany) 6.公司(Corporations) 7.跨国公司(Transnational Corporations) 第三章经营方式(Ways ofBusiness) 1.概述(Introduction) 2.批发(Wholesaling) 3.零售(Retailing) 4.特许经营(Franchising) 5.代理(Agency) 6.电子商务(E-merce)

2011年5月国际商务英语等级考试(中级)试卷及答案

全国外经贸从业人员考试 国际商务英语等级考试(中级)试卷 2011年5月 Ⅰ.Listening: 20%(听力,20分) Section A Directions:Listen carefully and fill in the blanks with the words or phrases you’ve heard. L/C is a written by a bank given to the exporter at the request of the importer to effect up to a stated sum of money within a prescribed time limit and against stipulated documents. L/C is the most generally used method of payment in the of international trade. It is reliable and safe because it gives protection to both the exporter and the importer. The exporter is assured of payment once he has shipped the goods as stipulated in the and the importer is assured of getting the right goods once he has made payment accordingly. Section B Directions: Listen to the tape and translate the sentences into Chinese. 1. . 2. . 3. . 国际商务英语等级考试(中级)试卷第1 页(共13页)

国际商务英语试题(一)

广东外语外贸大学公开学院辅导资料 国际商务英语试卷(一) 课程代码:05844 I. Translate the following words and expressions from English into Chinese (10%) 1.International business 2.GDP 3.Free trade area 4.Multinational corporation 5.Specialization 6.specific duty 7.roll on roll off 8.EDI 9.article number 10.auction Ⅱ. Translate the following words and expressions from Chinese into English (10%) 11.返倾销 12.最惠国条款 13.贴现 14.充分就业 15.保兑信用证 16.零库存 17.代位,取代 18.保护主义 19.免除条款(豁免条款) 20.证券交易所 Ⅲ.Match the words and expressions on the left with the explanations on the right (10%) 21.contracting party a. amount above what is estimated as necessary 22.gilts b. a business concern owned or controlled in whole or in part by another concern 23.cyclical c. recurring in cycles 24.affiliate d. stocks issued by government 25.legal action e. an action taken against some one in accordance with the law. 26.margin f. a country or firm that signs a legal agreement 27.pooling g. a trace of land including its buildings 28.verified data h. a set of international rules for the interpretation of

《国际商务英语》期中考试题

《国际商务英语》期中考试题 i. Put the following from English into Chinese or vice versa. 1. terms of payment 付款方法 6.预付运费Prepaid freight 2. enclosed please find 随函寄上7.报CIF伦敦价格Quote CIF London price 3. by separate post 另行邮寄8.即期信用证Sight L/C 4. profit margin 边际利润9. 以我方为受益人in our favor 5. quantity discounts 数量折扣10. 装船通知Shipping advice ii. Translate the following sentences from English into Chinese. 1.Your company has been introduced to us by the Bank of China as prospective buyers of electronic products. As we deal in the items, we shall be pleased to establish direct business relations with you. 中国银行向我们介绍贵公司是有希望的电子产品买主。由于我们经营该商品,我们很乐意与你建立直接的贸易关系。 2.In reply to your letter dated March 28, we enclose our latest illustrated catalogue for your reference. 兹复你方3月28日函,我们附上最新的带有插图的目录,供你方参考。 3.We are interested in your “Red Flower” Brand Varnish(清漆), but regret to say that your price is about 8% higher than that of other suppliers for similar quality products. 我们对你方的“红色的花”牌清漆(清漆)感兴趣,但很遗憾你方的价格高于其他供应商质量相同的产品价格8%左右。 iii. Translate the following sentences from Chinese into English. 1.如果你方价格有竞争力,我们愿首次订购1200打,即各种型号各400打。 If your prices are competitive,we are willing to place our first order for 1200 dozen,ie,400 dozen of each type. 2.我们的产品在世界市场上十分畅销,以其优良的品质和合理的价格而值得介绍。 Our products are popular in the world market, with its excellent quality and reasonable price worth introduction. 3.请报最好的CIF上海价格,注明最早装运期。 Please quote us your best CIF Shanghai price, indicating the earliest time of shipment. 4.最近对我们的山地自行车需求量很大,我们不能保证新订货在6月30日之前交付。 recently, mountain bike great demand for us, there is no guarantee that new orders on June 30 before delivery. 5.我们已决定委托你方在中国独家代理我方的玩具产品。 We've decided to entrust you with the sole agent of our toy products in China iv. Translate the following letter into English. XX 先生: 我们感谢你方1月2日的询盘,并高兴地获悉你方愿与我们建立贸易关系。按照要求,现向你方报盘如下,以你方答复于20XX年11月7日前到达此地为有效。 规格:蝴蝶牌浴巾(Turkish towel) NJ011 数量:20000打 价格:纽约到岸价含佣金5%,每打22美圆。 包装:每盒一打,每一纸板箱20盒。

(完整word版)国际商务函电考试大纲

一.英译汉 1. counter-signature 会签 2. We are sending you our S/C Number 200 in duplicate. 现特此寄上我方销售确认书第200号一式两份,其中一份请在签退后退回供我方存档。 3. Enclosed is our sales confirmation Number 35 in duplicate. Please sign a copy and return for our file. 随函附上我方销售确认书35号一式两份。请签署一份副本,并返回我们的文件。 4. Documentary L/C 跟单信用证 5. Extension of L/C 信用证延期 6. It appears that the stipulations in the L/C are not in agreement with the contract. We are cabling you this afternoon, asking you to amend the L/C to read: TRANSHIPMENT AND PARTSHIPMENTS ALLOWED 7. The price fixed at a reasonable level. 价格固定在一个合理的水平。 8. The price has advanced considerably. 价格大幅上涨。 9. be not in a position to make any further reduction. 不能再次降价。 10. If it were not for the friendship between us, we would not have made affirm offer at such a low price. 如果不是为了我们之间的友谊,我们就不会以如此低的价格来确认报价。 11. As the market is weak at present, your quotation is unworkable. 由于目前市场疲软,你方报价是行不通的。 12. shipping advice装船通知 13. partial shipment分批装运 14. initial orders初始订单 15. Please quote your lowest price C.I.F. Singapore for each of the following items, inclusive of our 3% commission. 请就下列每项货物向我方报成本加运费、保险费到新加坡的最低价格,其中包括我们百分之三的佣金。 16. It is stipulated by the contract that the seller should fax the shipping advice to the buyer upon making the consignment on board.

国际商务英语函电答案.doc

国际商务英语函电答案【篇一:世纪商务英语外贸函电(第二版)课后习题答案】 letter 1.at, of, with, in, for letter 2.from, into, with, of, to p40 https://www.wendangku.net/doc/a14930802.html,rming, interesting, dealing, sample, details, quality, prices, applied, items, inquiry unit 3 p55 letter 1.from, for, by, with, on letter 2.with, in, of, in, from p56 2.referring, established, cost, quality, opinion, responsibility, part, satisfied, information, decision unit 4 p71 letter 1. to, of, at, in, by letter 2.with, in, with, for, with p73 2.advertisement, leading, interested, details, dealers, line, market, replying, over, item unit 5 p88 letter 1.for, with, at, by, to letter 2.for, for, by, at, by/under p89 2.inquiring, quotation, receipt, subject, confirmation, discount, catalogue, brochure, separate, appreciate unit 6 p105 letter 1.of, on, in, with, to letter 2.of, in, by, for, at p106 2.offer, regret, price, sold, level, difference, transaction, counter-offer, samples, acceptance unit 7 p122 letter 1.to, of, of, in, for,

商务英语综合教程第一学期习题答案

Unit 1 Company Profile 1.Translate the following Chinese terms into English. candidate patent industry personnel registered trademark core competency domain name intended market emerging market headquarter multinational corporation Chief Executive Officer stakeholder 2.Translate the following English terms into Chinese. 股本,股金总额 以人为本的解决方案 精于心简于形 对客户的深入了解 工业革命 回收利用 3.Translation: 宝洁公司始创于1837年,是世界上最大的日用消费品公司之一。2007财政年度,公司全年销售额达682亿美元。在《财富》杂志评选出的全球500家最大企业中,排名第74位。 宝洁公司在全球80多个国家设有工厂或分公司,所经营的300多个品牌的产品畅销160多个国家和地区,其中包括美容护理、居家护理、吉列产品等。每天,在世界各地,宝洁公司的产品与全球消费者发生着三十亿次亲密接触。宝洁大中华业务区包括1988年成立的中国大陆分公司、1987成立的香港分公司和1985年成立的台湾分公司。 一九八八年,宝洁公司在广州成立了在中国的第一家合资企业-广州宝洁有限公司,从此开始了其中国业务发展的历程。宝洁总部位于广州,目前在广州、北京、上海、成都、天津、东莞及南平等地设有多家分公司及工厂,并在北京设立技术中心。 二十年来,宝洁取得了飞速的发展,主要表现在:

外贸函电试卷及答案汇编

Final Paper Part One: Explain the following terms in appropriate English or Chinese.(16points) 1.信头包括: 2. 2. 商务信函的写作原则: 3.3. 订单一般包括的内容: 4.4. 保险索赔的条件 Part Two: Please fill in the blanks with appropriate words.(20 points) 1.As we are interested ____ Art. No. 503, please make us a firm offer for 1000 pieces. 2.We ______ from the Commercial Counselor's Office in your country that you are one of the large buyers of color TV sets . 3.We shall be glad to send you samples upon receipt _____ your specific enquiries . 4.Electric goods ______ within our scope of business. 5.Please let us have all necessary information concerning your products ____ exports. 6.We shall be _____ to establish business relations with your company. 7.As we deal ____ sewing machines, we shall be pleased to enter into business relations with you. 8. We are keenly desirous _____ entering into business relations with your firm. 9.On receipt of your specific inquiry, we shall airmail you immediately our _____ sheet. 10.We write to ____ ourselves as one of the leading exporters of a wide range of electronic goods in the UK. 11.We would prefer FOB Lagos, introducing our _____ of 8%. 12.Any _____ you supply will be highly appreciated. 13.Several of our customers have expressed ____ in your watches and inquired about their quality and prices. 14.One of our buyers has made us an enquiry _____ 150 dozen men's shirts. 15.If your samples are satisfactory _____ our customers, we will place substantial orders with you. 16.Your prompt attention_____ this inquiry is appreciate. 17.We have a considerable demand here ______ your goods and should welcome your samples. 18. Will you please quote us your lowest prices ______ the goods listed above? 19.We would very much ______ an early reply from you. 20.Please _____ us your competitive price for the following items. Part Three :Judge whether the following statements are true or false with V or X. (10 points) 1.外贸函电的日期应写在信函最后。() 2.缩进式英文信函格式最不正式。() 3.如果知道收信人的名字,最好称呼他们的头衔或姓。() 4.英文信函只有打印签名即可。() 5.英文信函的结束语一般不用标点。() 6.当你方因某种原因不能接受订单时,你可以采用送上代用品的方法。() 7.在进出口业务中只能是买方向买房提出索赔要求。() 8.常见的国际贸易支付方式只有汇付和托收。( ) 9.拒绝订货信要写得委婉,解释清楚原因,并要表示对将来生意的期盼。( )

国际商务函电课后习题答案

CHAPTER ONE ESTABLISHING BUSINESS RELATIONS III. Translate the following sentences into English. 1. Our company has various kinds of Men’s Shirts for export. 2. We are one of the largest exporters of Leather Goods in this area. 3. As requested, we are sending you under separate cover the latest catalogue for your reference. 4. We shall be glad to establish business relations with you if you are interested in Sweaters. 5. Having obtained your name and address from Tokyo ABC Company , we are writing you in the hope of establishing business relations with you. V. Write a letter of establishing business relations according to the following situation. Dear Sirs, We obtain you name and address from the internet, and we are writing you in the hope of establishing direct business relations with you. We’re one of the importers of Beddings with years’ of experience in this line. At present, we’ll be pleased to get the samples for your bed-sheets. Thank you for your cooperation. Yours sincerely, Lesson Two IV. Translate the following letter into English. Dear Sirs, Your company has been introduced to us by FMC Company, Sydney, Australia,who has informed us that you are interested in the electric goods . As we have been in this line for years, we are writing you in the hope of establishing business relations with you. In order to give you a rough idea of our products available for export at present, we are sending you under separate cover the latest catalogues. Looking forward to your early reply. Yours faithfully, V. Write a letter according to the following situation. Dear Sirs, We learn from the internet that you are in urgent need of a large quantity of Children’s Sport Shoes. We’re writing you in the hope of entering into long-term business relations with you. We have been handling the export of various kinds of textiles and shoes for many years, and our products are very popular in the American and European markets for their good quality and reasonable prices. We also have kept close connections with the local shoe manufacturers, and can ensure the steady source and quality. In order to give you a rough idea of our products, we are sending you our latest catalogues and price lists for Children’s Sport Shoes for your reference. If you are interrested in any of the items, please let us know. We await your early reply. Yours faithfully, Lesson Three III. Tranlate the following sentences into English. 1. We are desirous of entering into direct business relations with you. 2. We are in a position to supply you with various kinds of canned food at the best prices. 3. Please supply us with the detailed information regarding these goods. 4. We are in a position to supply you with a large quantity of Color TVs at competitive prices. 5. We hope to meet your requirements. IV. Write letters in proper forms according to the following particulars. A: Messrs. Anderson & Co. 17 Mayfield Road, Copenhagen, Denmark December 12 Tianhong International Trading Company Limited Rm.1202 Yinze Manson, Huangpu Road, Guangzhou, China

商务英语函电试卷A 《商务英语函电》试卷

商丘师范学院2012——2013学年度第二学期期终考试 国际教育学院 商务英语专业11级专科《商务英语函电》试卷 说明:本试卷共4页、五道大题,答卷一律在试卷内规定处进行,答在其它处不得分。 PART I BLANKS (10%. 2 points for each blank) Fill in the blanks with a proper English words or phrases. 1. What does the term “CIF Shanghai ” mean in Chinese? _________________________________ 2. When the buyer draft the letter of partial rejection, he should cover the following point: a._______________________________________________ b._______________________________________________ c._______________________________________________ d._______________________________________________ PART II MULTIPLE CHOICE (10%. 1 points for each) There are 10 questions in this section. Choose the best answer for each question. 6. We _______ you of our prompt shipment after receipt of your order. A. assure B. assurance C. insure D. insurance 7. We have to claim for your delay in _______ of our order. A. shipping B. shipment C. ship D. deliver 8. Our company has wide experience ________ the textiles line. A. on B. in C. about D. of 9. You have delayed in dispatching us the shipping _______ after shipment. A. advice B. instruction C. information D. data 10. Please effect payment when the draft _______ . A. expires B. due C. falls due D. falls expiration 11. Our latest design has won worldwide ________. A. popular B. popularity C. popularly D. popularize 12. Art. No.8905 enjoys the fame of wide ________. A. selling B. salable C. sales D. sell 13. We are unable to satisfy your requirements, for the goods are _______ great demand.

相关文档
相关文档 最新文档